Problem Set 3 2
Problem Set 3 2
Introduction
This problem set contains 6 problems on functional equations.
Instructions:
• You can discuss the problems with one another and come up with solutions together
as well. This is especially encouraged for harder problems. We would request you
to solve the easier ones independently.
• You are also encouraged to ask for hints on the server on problems you’re struggling
with.
• The solutions/sources of the problems will be posted with the next Pset.
• Problems are roughly ordered by difficulty.
• Enjoy the problems!
1
Online Math Club (May 17, 2024) Problem Set 3
§1 Problems
Problem 1.1. Find all functions f : R → R (from the set of real numbers to itself)
where
f (x − y) + xf (x − 1) + f (y) = x2
for all reals x, y.
Problem 1.3. Let Z be the set of integers. Determine all functions f : Z → Z such that,
for all integers a and b,
f (2a) + 2f (b) = f (f (a + b))
for all x, y ∈ R
2
Online Math Club (May 17, 2024) Problem Set 3
§2 Solutions/Sources to Pset 2
Problem 2.1. You have to find 10321032 (mod 100).
Notice that
10321032 ≡ 0 (mod 4) and 10321032 ≡ 1 (mod 25)
So, by CRT, or otherwise we can combine these two to get that
Problem 2.2. The easiest thing is to find an x ≡ c (mod p) which satisfies the given
condition as well.
If x ≡ 1 (mod p − 1) then we are done. Both of these modular equations can be combined
by CRT and that proves the existence of such an x for every c, p.
Problem 2.3. AIME II 2012 Problem 12
Problem 2.4. IMO Shortlist 2005 N6
Problem 2.5. For a prime p ̸= 11, let m = pa b, n = pc d where b and d are relatively
prime to p. If we prove a = c for any prime p ̸= 11, then we are done.
WLOG, let a > c.
We know there exists a solution for k such that 11k ≡ 1 (mod pa ). For such a k, we have
pa | gcd(11k − 1, m)
=⇒ pa | gcd(11k − 1, n)
=⇒ pa | n =⇒ pa | pc =⇒ a ≤ c
contradicting the assumption that a > c. Hence we are done.
Problem 2.6. Let the points on the grid be of the form
(x, y) = (a + m, b + n), 0 ≤ m, n, ≤ 99
Let {pj } be a sequence
Q100 of distinct prime numbers.
Let a ≡ 0 (mod i=1 pi ). Then let,
b ≡ 0 (mod p1 )
b + 1 ≡ 0 (mod p )
2
...
b + 99 ≡ 0 (mod p )
100
3
Online Math Club (May 17, 2024) Problem Set 3
and by CRT we know that a solution exists. Therefore, we have proven the existence of
a 100 × 100 grid such that none of the points on it are visible.
Problem 2.7. Let p be a prime that does not divide any element of S. Consider the set
R = {r1 , r2 , . . . , rk } which is the set of all possible remainders when an element of S is
divided by p. We know that 0 ̸∈ R.
We are given that a, b ∈ S =⇒ ab + 1 ∈ S
≡ ri , rj ∈ R =⇒ ri rj + 1 ∈ R
=⇒ {r1 , . . . , rk } = {r1 r1 + 1, r1 r2 + 1, . . . , r1 rk + 1}
Therefore, the sum of all elements in these two sets must be the same.
=⇒ r1 + . . . + rk = r1 (r1 + . . . + rk ) + k
Now, consider a prime p > x, y where x, y are the smallest two elements of S. Clearly
from the above argument, p can’t be the prime which doesn’t divide any element of S.
So the only primes which potentially work are bounded above by max(x, y) and is thus
finite.
Hence proved.
Problem 2.8. Folklore. An even number of odd numbers sum to an even number so it
is not possible
Problem 2.9. Note that each person sees 2 other hats. Consider the following strategy:
if you see opposite colors, skip. If you see the same color twice, guess the color opposite
to the one you see. This works as long as not all three had colors are the same, ie with
6/8 probability.
Problem 2.10. We use induction on n. The base case of n = 1 is clear. If n ever reaches
the end, note that it stays there, so only the first n − 1 numbers matter and we can
use the inductive hypothesis. If n never reaches the end, this means that it could never
have appeared at the start of the sequence. In particular, this means that n was never
operated on. Let the last number at a given moment be k. The only number that can
displace k is n so we can conclude that k must stay at the end as well. In particular, this
means that neither n nor k could have ever reached the end. Let’s assume that for the
sake of contradiction there is some such arrangement so that 1 never reaches the start.
Note that even if we exchange the positions of the n and k, the same operations should
take place and 1 will never reach the start. But this is a contradiction to the inductive
hypothesis since n is at the end in the new sequence